Argument Structure Questions - - Question 18

Economist:  To the extent that homelessness arises from a lack of available housing, it should not be assumed that th...

KDA86 July 21, 2014

Please explain

Please explain why (A) is correct.

Replies
Create a free account to read and take part in forum discussions.

Already have an account? log in

Naz July 21, 2014

Here we have an Argument Structure question. The question stem of the argument will refer us to a specific line or phrase. We are tasked to find the answer choice that correctly describes the role of the referenced portion of the argument.

The conclusion of the argument is: "it should not be assumed that the profit motive is at fault." Why? "It is unrealistic to expect investors to take risks with their property unless they get some benefit in return." Private investors will provide housing as long as the market permits them to make a profit.

Now, why is the Economist focusing on the housing market and property investors? Because of the following phrase: "To the extent that homelessness arises from a lack of available housing."

This is the phrase that the question stem refers us to. It places a restriction on the argument to only focus on homelessness arising out of a lack of available houses, as opposed to, for example, homelessness arising out of drugs or job layoffs.

Thus, this phrase limits the focus of the argument to one part of the issue of homelessness, i.e. the lack of available housing. Therefore, answer choice (A): "It limits the application of the argument to a part of the problem," is our correct answer.

Hope that was helpful! Please let us know if you have any other questions.

Sallia91 April 1, 2015

Why is A correct?

Naz April 1, 2015

If you click on the other thread in this question, it will open to my previous explanation.

Hope that helps! Please let us know if you have any other questions.